Improper integral question

Since the original question involves taking the arctan, it is reasonable to assume that the value of x is small enough to use this approximation.
  • #1
TsAmE
132
0

Homework Statement



Determine how large the number a has to be so that:

[tex]\int_{a}^{\infty} \frac{1}{1 + x^{2}} dx < 0.001[/tex]

Homework Equations



None.

The Attempt at a Solution



I tried to evaluate the left hand side and got a final answer of:

[tex]a > \frac{\pi}{2} - \frac{1}{1000}[/tex]

but the correct answer was 1000:confused:

I think these 2 steps in my working out might be the problem:

from [tex]tan(arctan(a)) > \frac{\pi}{2} - \frac{1}{1000}[/tex] to:

[tex]arctan(a) > \frac{\pi}{2} - \frac{1}{1000}[/tex]

[tex]a > \frac{\pi}{2} - \frac{1}{1000}[/tex]
 
Last edited:
Physics news on Phys.org
  • #2
Why did you write tan(arctan(a))? Since the indefinite integral is just arctan(x), and arctan(x) ->pi/2 as x->infinity, you should just write pi/2-arctan(a)<.001, or arctan(a)>pi/2-.001. Do you know how to solve this?
 
  • #3
The last 3 lines do not continue from each other at all. Firstly, how did you get tan(arctan(a))? How does this continue on to get just arctan(a) and then simply a?
 
  • #4
Sorry I made a mistake in the post. I have corrected the question.
 
  • #5
My last post was ambiguous. I meant I had corrected the mistake when I typed the question. Could someone please check my working?
 
  • #6
Perform the integral to obtain:
[tex]
\int_{a}^{\infty}\frac{dx}{1+x^{2}}=\tan^{-1}(\infty )-\tan^{-1}a=\frac{\pi}{2}-\tan^{-1}a<0.001
[/tex]
Then re-arrange to obtain:
[tex]
\tan^{-1}a>\frac{\pi}{2}-0.001
[/tex]
Take tan to obtain the answer...
 
  • #7
TsAmE said:
My last post was ambiguous. I meant I had corrected the mistake when I typed the question. Could someone please check my working?

I don't see any change in your original post. Your last 3 lines of working are still wrong.
hunt_mat has given the answer.
 
  • #8
Mentallic said:
I don't see any change in your original post. Your last 3 lines of working are still wrong.
hunt_mat has given the answer.

I reached the same answer as hunt as in:

arctan of [tex]> \frac{\pi}{2} - \frac{1}{1000}[/tex]

and tried solving for a only to get:

[tex] a > \frac{\pi}{2} - \frac{1}{1000}[/tex]

I don't see how I can get an answer of 1000 when there is a pi/2 - 0.001

My latex code is giving me problems when I try to edit it so that's probably why you didnt see my correction.
 
Last edited:
  • #9
So take tan of what I wrote down:
[tex]
a>\tan\Bigg(\frac{\pi}{2}-\frac{1}{1000}\Bigg) =\frac{1}{\tan (1/1000)}\approx 1000
[/tex]
That is how you finish off the calculation.
 
  • #10
hunt_mat said:
So take tan of what I wrote down:
[tex]
a>\tan\Bigg(\frac{\pi}{2}-\frac{1}{1000}\Bigg) =\frac{1}{\tan (1/1000)}\approx 1000
[/tex]
That is how you finish off the calculation.

With respect to the inequality why is a > and not < ? Do you swap the sign around when you use the inverse trig function?

Also [tex]a>\tan\Bigg(\frac{\pi}{2}-\frac{1}{1000}\Bigg) =[/tex] 0.0274... on my calculator as opposed to 1000?:confused:
 
  • #11
You use:
[tex]
\tan x=\frac{\sin x}{\cos x}
[/tex]
And then you note:
[tex]
\sin\Bigg(\frac{\pi}{2}-\frac{1}{1000}\Bigg) =\cos\Bigg(\frac{1}{1000}\Bigg)
[/tex]
and
[tex]
\cos\Bigg(\frac{\pi}{2}-\frac{1}{1000}\Bigg) =\sin\Bigg(\frac{1}{1000}\Bigg)
[/tex]
So you get the same as before, for small x, we have the following:
[tex]
\tan x\approx x
[/tex]
Regarding the inequality, tan x is an increasing function, so the inequality remains the same, regarding your calculator, I think you may have entered the sum incorrectly.
 
  • #12
TsAmE said:
With respect to the inequality why is a > and not < ? Do you swap the sign around when you use the inverse trig function?
I don't understand why you think it would be < ?
You already had [itex]arctan(a)>\pi/2-0.001[/itex] so why would it suddenly change sign?

TsAmE said:
Also [tex]a>\tan\Bigg(\frac{\pi}{2}-\frac{1}{1000}\Bigg) =[/tex] 0.0274... on my calculator as opposed to 1000?:confused:

Your calculator is set in degrees and you want radians.
 
  • #13
Thanks I now see where I went wrong. One last thing I want to know is how would you know that [tex]\frac{1}{\tan (1/1000)}\approx 1000[/tex] without using a calculator, cause I am not allowed to use calculators in my tests.
 
  • #14
Use a Maclaurin series to show that:
[tex]
\tan x =x+o(x),\quad \sin x=x+o(x),\quad\cos x=1-\frac{x^{2}}{2}+o(x^{2})
[/tex]
 
  • #15
TsAmE said:
Thanks I now see where I went wrong. One last thing I want to know is how would you know that [tex]\frac{1}{\tan (1/1000)}\approx 1000[/tex] without using a calculator, cause I am not allowed to use calculators in my tests.

For small x, [itex]x\approx tan(x)[/itex] so [tex]\frac{1}{tan(1/1000)}\approx \frac{1}{1/1000}=1000[/tex]
 

What is an improper integral?

An improper integral is an integral where either the upper or lower limit of integration is infinite, or where the function being integrated is undefined at some point within the interval of integration.

How do you determine if an improper integral converges or diverges?

To determine if an improper integral converges or diverges, you can use various tests such as the comparison test, limit comparison test, or the p-series test. These tests compare the improper integral to a known convergent or divergent series or integral.

What is the difference between a type 1 and type 2 improper integral?

A type 1 improper integral has one or both limits of integration at infinity, while a type 2 improper integral has a function that is undefined at some point within the interval of integration.

Can an improper integral have both limits of integration at infinity?

Yes, an improper integral can have both limits of integration at infinity. In this case, the integral is called a double improper integral.

Is it possible for an improper integral to have a finite value even if the function being integrated is not defined at some point within the interval of integration?

Yes, it is possible for an improper integral to have a finite value even if the function being integrated is not defined at some point within the interval of integration. This can occur if the function has a removable discontinuity at that point.

Similar threads

  • Calculus and Beyond Homework Help
Replies
3
Views
327
  • Calculus and Beyond Homework Help
Replies
5
Views
1K
  • Calculus and Beyond Homework Help
Replies
7
Views
923
  • Calculus and Beyond Homework Help
Replies
9
Views
127
  • Calculus and Beyond Homework Help
Replies
3
Views
226
  • Calculus and Beyond Homework Help
Replies
9
Views
738
  • Calculus and Beyond Homework Help
Replies
1
Views
323
  • Calculus and Beyond Homework Help
Replies
4
Views
94
  • Calculus and Beyond Homework Help
Replies
3
Views
397
  • Calculus and Beyond Homework Help
Replies
8
Views
646
Back
Top